Last visit was: 24 Apr 2024, 13:37 It is currently 24 Apr 2024, 13:37

Close
GMAT Club Daily Prep
Thank you for using the timer - this advanced tool can estimate your performance and suggest more practice questions. We have subscribed you to Daily Prep Questions via email.

Customized
for You

we will pick new questions that match your level based on your Timer History

Track
Your Progress

every week, we’ll send you an estimated GMAT score based on your performance

Practice
Pays

we will pick new questions that match your level based on your Timer History
Not interested in getting valuable practice questions and articles delivered to your email? No problem, unsubscribe here.
Close
Request Expert Reply
Confirm Cancel
SORT BY:
Kudos
Tags:
Difficulty: 705-805 Levelx   Percent and Interest Problemsx                              
Show Tags
Hide Tags
Tutor
Joined: 04 Aug 2010
Posts: 1315
Own Kudos [?]: 3136 [2]
Given Kudos: 9
Schools:Dartmouth College
Send PM
GMAT Club Legend
GMAT Club Legend
Joined: 03 Oct 2013
Affiliations: CrackVerbal
Posts: 4946
Own Kudos [?]: 7626 [2]
Given Kudos: 215
Location: India
Send PM
User avatar
Manager
Manager
Joined: 28 Dec 2013
Posts: 50
Own Kudos [?]: 14 [1]
Given Kudos: 3
Send PM
Tutor
Joined: 16 Oct 2010
Posts: 14817
Own Kudos [?]: 64899 [1]
Given Kudos: 426
Location: Pune, India
Send PM
Re: A photography dealer ordered 60 Model X cameras to be sold [#permalink]
1
Kudos
Expert Reply
sagnik242 wrote:
VeritasPrepKarishma wrote:
pgmat wrote:
A photography dealer ordered 60 Model X cameras to be sold for $250 each, which represents a 20 percent markup over the dealer’s initial cost for each camera. Of the cameras ordered, 6 were never sold and were returned to the manufacturer for a refund of 50 percent of the dealer's initial cost. What was the dealer's approximate profit or loss as a percent of the dealer’s initial cost for the 60 cameras?

A. 7% loss
B. 13% loss
C. 7% profit
D. 13% profit
E. 15% profit


Use weighted avgs for a quick solution:

On 54 cameras, the dealer made a 20% profit and on 6 cameras, he made a 50% loss. Cost Price of 54 cameras:cost price of 6 cameras = 54:6 = 9:1

Avg Profit/Loss = (.2*9 + (-.5)*1)/10 = 1.3/10 = .13 = 13% profit

For more on weighted avgs, check: https://www.gmatclub.com/forum/veritas-prep-resource-links-no-longer-available-399979.html#/2011/03 ... -averages/

why are we dividing by 10?


Weighted Average Formula:

Cavg = (C1*w1 + C2*w2)/(w1 + w2)

C1 and C2 represent the quantity which we want to average so they will be profit/loss here.
C1 = 20% = .2
C2 = -50% (loss) = -.5
The weights, given by w1 and w2, are the cost prices.
Cost Price of 54 cameras:cost price of 6 cameras = 54:6 = 9:1
So w1 =9 and w2 = 1

Avg Profit/Loss = (.2*9 + (-.5)*1)/(9 + 1) = (.2*9 + (-.5)*1)/10

Get more details on this concept from the link given in my post above.
e-GMAT Representative
Joined: 04 Jan 2015
Posts: 3726
Own Kudos [?]: 16832 [1]
Given Kudos: 165
Send PM
Re: A photography dealer ordered 60 Model X cameras to be sold [#permalink]
1
Kudos
Expert Reply
Andrewbpaa wrote:
Suppose: what if I want to calculate the number of the total profit first. How can I get it? I tried to get it by "54^250/1.2. Is this is the correct way? Pls enlighten. Thanks in advance.


Hi Andrewbpaa,

For calculating the total net profit, you would need to know the cost price and the selling price of all the 60 cameras:

On a selling price of $250, the dealer makes a profit of 20%. So, his cost price for 1 camera would be \(= \frac{$250}{1.20}\)
His refund price for 6 cameras is 50% of his cost price i.e. \(0.5* \frac{$250}{1.20}\)

I. Profit from selling 54 cameras at $250 each\(= 54 * ( $ 250 - \frac{$ 250}{1.20})\)
II. Loss from refund of 6 cameras = \(6 *( \frac{$250}{1.20} - \frac{$ 250}{1.20} * 0.50)\) \(= 6*( \frac{$250}{1.20} * 0.50)\)

Adding I & II would give you the total net profit the dealer made on the 60 cameras. The net profit can then be divided by the total cost price of 60 cameras to arrive at the total profit% of the dealer.

However, I would suggest you use weighted average method to solve the question.

Points to Note
In such questions, it pays to be careful about the concept of markup% and discount%. In this question, there was no discount offered, hence the markup price and selling price were the same i.e. $250. Had there been a discount of lets say 10%, the selling price would have been \($250* 0.9 = $ 225\). The profit then would have been equal to \($225 - \frac{$ 250}{1.20}\)

Please remember that the markup% is calculated on the cost price while the discount % is calculated on the markup price.

Given below are few questions on discount & markup for your practice:

if-the-original-price-of-an-item-in-a-retail-store-is-marked-163706.html?hilit=discount
a-pair-of-skis-originally-cost-160-after-discount-of-x-149918.html
a-jewelry-dealer-initially-offered-a-bracelet-for-sale-at-an-8215.html?hilit=discount%20&%20markup
henry-purchased-three-items-during-a-sale-he-received-a-20-discount-194280.html?hilit=discount

Hope its clear!

Regards
Harsh
avatar
Intern
Intern
Joined: 16 Aug 2015
Posts: 17
Own Kudos [?]: 62 [1]
Given Kudos: 15
Location: India
Concentration: General Management, Entrepreneurship
GMAT 1: 510 Q22 V19
GPA: 3.41
Send PM
Re: A photography dealer ordered 60 Model X cameras to be sold [#permalink]
1
Bookmarks
A big question I have is, why does following method give different answer though I believe there is no flaw in my method of attempting this problem -

Purchase price per camera = 250/1.2 = 1250/6.

Total purchase price = 60 * 1250/6 = 12500$

Got return money because of 6 cameras which were returned = 50% of 6 = [(6 * 250/1.2)/2] = 625$

Effective purchase price = 12500 - 625 = 11875$

Total sale = 54 * 250 = 13500$.

Hence % profit = [(Total sale - effective purchase price) / (effective purchase price)] x 100

= [(13500 - 11875) / 11875] x 100

= (1625/11875) x 100

= 13.684 Ans.

Dealer paid total cost of 60 cameras and then I deducted 50% cost of 6 cameras from that instead of adding it as a revenue. So in this case, technically 54 cameras have been sold and 57 were bought.

Can anyone explain why can't I do that way?

TIA
Tutor
Joined: 16 Oct 2010
Posts: 14817
Own Kudos [?]: 64899 [1]
Given Kudos: 426
Location: Pune, India
Send PM
Re: A photography dealer ordered 60 Model X cameras to be sold [#permalink]
1
Kudos
Expert Reply
bablu1234 wrote:
A big question I have is, why does following method give different answer though I believe there is no flaw in my method of attempting this problem -

Purchase price per camera = 250/1.2 = 1250/6.

Total purchase price = 60 * 1250/6 = 12500$

Got return money because of 6 cameras which were returned = 50% of 6 = [(6 * 250/1.2)/2] = 625$

Effective purchase price = 12500 - 625 = 11875$

Total sale = 54 * 250 = 13500$.

Hence % profit = [(Total sale - effective purchase price) / (effective purchase price)] x 100

= [(13500 - 11875) / 11875] x 100

= (1625/11875) x 100

= 13.684 Ans.

Dealer paid total cost of 60 cameras and then I deducted 50% cost of 6 cameras from that instead of adding it as a revenue. So in this case, technically 54 cameras have been sold and 57 were bought.

Can anyone explain why can't I do that way?

TIA


You have assumed the situation to be a bit different from what it actually is.

You have assumed that he bought 54 cameras at 250/1.2 each and 6 cameras at 125/1.2 each.
Then he sold 54 cameras at 250 each and 6 cameras at 0.

While actually this is the situation:

He bought 60 cameras at 250/1.2 each.
He sold 54 at 250 each and sold the rest of the 6 at 125/1.2 each.

The difference (Revenue - Cost) in both cases will be the same but the cost price in the denominator will be different. So profit % obtained will be different. Please check Bunuel's solution on Page 1 to see the actual calculations of this method.
Tutor
Joined: 16 Oct 2010
Posts: 14817
Own Kudos [?]: 64899 [1]
Given Kudos: 426
Location: Pune, India
Send PM
Re: A photography dealer ordered 60 Model X cameras to be sold [#permalink]
1
Kudos
Expert Reply
sagnik242 wrote:
VeritasPrepKarishma wrote:
pgmat wrote:
A photography dealer ordered 60 Model X cameras to be sold for $250 each, which represents a 20 percent markup over the dealer’s initial cost for each camera. Of the cameras ordered, 6 were never sold and were returned to the manufacturer for a refund of 50 percent of the dealer's initial cost. What was the dealer's approximate profit or loss as a percent of the dealer’s initial cost for the 60 cameras?

A. 7% loss
B. 13% loss
C. 7% profit
D. 13% profit
E. 15% profit


Use weighted avgs for a quick solution:

On 54 cameras, the dealer made a 20% profit and on 6 cameras, he made a 50% loss. Cost Price of 54 cameras:cost price of 6 cameras = 54:6 = 9:1

Avg Profit/Loss = (.2*9 + (-.5)*1)/10 = 1.3/10 = .13 = 13% profit

For more on weighted avgs, check: https://www.gmatclub.com/forum/veritas-prep-resource-links-no-longer-available-399979.html#/2011/03 ... -averages/

why are we dividing by 10?


The weights are 54 (20% profit on 54 cameras) and 6 (50% loss on 6 cameras).
This is a ratio of 54:6 = 9:1 (in lowest terms). It is the same thing whether you use 54 and 6 as weights or 9 and 1 but 9 and 1 simplify the calculations.

Avg = (A1 * w1 + A2 * w2)/(w1 + w2) = (A1 * 9 + A2 * 1)/(9 + 1) = (A1 * 9 + A2 * 1)/10
Tutor
Joined: 16 Oct 2010
Posts: 14817
Own Kudos [?]: 64899 [1]
Given Kudos: 426
Location: Pune, India
Send PM
Re: A photography dealer ordered 60 Model X cameras to be sold [#permalink]
1
Kudos
Expert Reply
philipb wrote:
Does this question bother anyone else in formatting terms? Perhaps I'm misinterpreting, but let me explain:

"What was the dealer's approximate profit or loss as a percent of the dealer's initial cost for the 60 cameras?"

Profit = Revenue - Costs

Thus, the question is looking for (Revenue - Cost) / Cost

In number terms this would look like, [54*(250) + 6*(250)*(4/5)*(1/2) - 600*(250)*(4/5)]/[(60)*(250)*(4/5)]x100 ~~ 10.7%

I'd appreciate any help in clearing up this misunderstanding!


You are correct conceptually.

The error lies here:

"$250 each, which represents a 20 percent markup over the dealer’s initial cost for each camera."

Marked price is 250
This is a 20% markup on cost.

Cost * (1 + 20/100) = Marked price

Cost * (6/5) = Marked Price
Cost = (5/6) * 250

You have taken this to be (4/5)*250. Replace it by (5/6) and you should get the correct answer.
Tutor
Joined: 16 Oct 2010
Posts: 14817
Own Kudos [?]: 64899 [1]
Given Kudos: 426
Location: Pune, India
Send PM
Re: A photography dealer ordered 60 Model X cameras to be sold [#permalink]
1
Kudos
Expert Reply
susheelh wrote:
Hello diegocml,

I hope this is what you meant by scale method (See attachment).

Based on the scale we get

\(\frac{54}{6} = \frac{(x+50)}{(20-x)}\)

\(\frac{9}{1} = \frac{(x+50)}{(20-x)}\)

\(180 - 9x = x+50\)

\(130 = 10x\)

\(x = 13\)

So, the dealer earns a profit of 13%

I learnt this approach after reading VeritasPrepKarishma's blog. Since the answer is correct, I am assuming the approach is correct :)






This formula is an application of the scale method:

\(\frac{w1}{w2} = \frac{A2 - Aavg}{Aavg - A1}\)

It is faster and more direct than actually using the number line.
Intern
Intern
Joined: 26 Oct 2019
Posts: 10
Own Kudos [?]: 111 [1]
Given Kudos: 13
Location: United States (TX)
GPA: 3.93
WE:Sales (Manufacturing)
Send PM
Re: A photography dealer ordered 60 Model X cameras to be sold [#permalink]
1
Kudos
My thought process with very similar with akang. After reading the problem, I just saw "percent", "percent" "What percent...?" So I didn't even bother with the marked-up "$250"

In my head, I saw:

(6 out of 60) = 10% of stuff (never sold, returned, = loss) at 50% loss

(rest of stuff, not 6, minus 10%) = 90% of stuff (gives him 20% markup) at 20% profit

And his T (total) whatever percentage is Profit - Loss

So, my scratch paper for this problem literally looks like:

90% of 20% = 18%
10% of 50% = 5%
P - L = ___________

18% - 5% = 13%

Then I thought, "profit" since the gain was bigger than the loss.

The other clue to ditch the $250 and any other non-percentages was when I tried to figure out the actual cost per camera. When I tried to divvy up 250 by 1.2 and started to get something funky, I thought, "Ok, they don't want me to mess with that."

Right? So is my thought process applicable to other problems like this? We don't actually have to mess with actual total profit and dividing by huge numbers to get the answer, huh? Or am I narrowing my train of thought down too much that it will only work for this problem? And I totally feel you on the weighted averages equation, but is it a good habit to use weighted averages for all problems like this? Do we need the full meal deal, or can I get away with just the burger?

I just think it's funny when I see contributors online write out work like: "1,644/12,480 = ~13%. See?" As if were all supposed to be like, "Oh, of course! Divided by twelve thousand four hundred and eighty IS about thirteen percent. Makes total sense." Like us normal people could just come up with that answer that easily and that quickly, without a calculator, during the actual test. No offense to anyone, I love this site, but I already have the answers in the back of the book. What helps, at least, helps me, is when I can be shown how to break the work down into small digestible pieces that can be used on other questions as well. Like eating cookies in bed. The trick is to break them up inside the bag, then put the small cookie pieces in your mouth. No crumbs all over the sheets.

But maybe that's just me.

Thanks everyone.
Tutor
Joined: 16 Oct 2010
Posts: 14817
Own Kudos [?]: 64899 [1]
Given Kudos: 426
Location: Pune, India
Send PM
Re: A photography dealer ordered 60 Model X cameras to be sold [#permalink]
1
Kudos
Expert Reply
pgmat wrote:
A photography dealer ordered 60 Model X cameras to be sold for $250 each, which represents a 20 percent markup over the dealer’s initial cost for each camera. Of the cameras ordered, 6 were never sold and were returned to the manufacturer for a refund of 50 percent of the dealer's initial cost. What was the dealer's approximate profit or loss as a percent of the dealer’s initial cost for the 60 cameras?

A. 7% loss
B. 13% loss
C. 7% profit
D. 13% profit
E. 15% profit


Responding to a pm - a useful tip for everyone:

Quote:
In this question, the sold price of each camera is 250$ after 20% markup price. So i tried to find the original cost using the method 250 * .80 which calculates to 200 and 20% is just 240 but not 250. I have also tried with other numbers as well such as 250*.75, 90 but i am sure, am missing something :( .

Could you please help me understand, how should I do the reverse percentage calculation?


Don't try to calculate the reverse percentage lest you make a mistake. Use the straight forward percentage manipulation even if that needs a variable. Even if you do not want to actually write it down, make a straight forward equation in your mind like this:

Cost * (6/5) = 250

So
Cost = 250 * (5/6)
Manager
Manager
Joined: 13 Jul 2019
Posts: 51
Own Kudos [?]: 143 [1]
Given Kudos: 13
Send PM
Re: A photography dealer ordered 60 Model X cameras to be sold [#permalink]
1
Bookmarks
This question can be solved using the concept of weighted average.

Good cameras = 54 (Profit of 20%)
Bad cameras = 6 (Loss of 50%)
Attachments

Capture.PNG
Capture.PNG [ 3.41 KiB | Viewed 3229 times ]

Tutor
Joined: 26 Jun 2014
Status:Mentor & Coach | GMAT Q51 | CAT 99.98
Posts: 452
Own Kudos [?]: 759 [1]
Given Kudos: 8
Send PM
A photography dealer ordered 60 Model X cameras to be sold [#permalink]
1
Bookmarks
Expert Reply
pgmat wrote:
A photography dealer ordered 60 Model X cameras to be sold for $250 each, which represents a 20 percent markup over the dealer’s initial cost for each camera. Of the cameras ordered, 6 were never sold and were returned to the manufacturer for a refund of 50 percent of the dealer's initial cost. What was the dealer's approximate profit or loss as a percent of the dealer’s initial cost for the 60 cameras?

A. 7% loss
B. 13% loss
C. 7% profit
D. 13% profit
E. 15% profit


Method 1: - The long way:

SP of each camera is 120% of the cost
=> Cost of each camera = $(250/1.2) = $(1250/6)
Total cost = (1250/6) x 60 = $12500
SP from 54 cameras = 250 x 54 = $13500
The remaining 6 cameras were returned for half the cost price, i.e. (1250/6)/2 = $(625/6)
Price obtained from these 6 = 6 x 625/6 = $625
Total SP = 13500 + 625 = $14125
Profit % = (14125 - 12500)/12500 x 100 = 13%

Method 2: - The short way:

In questions that ask for percent profit, the actual values aren't required - we just need to know the ratio of the prices and the ratio of the quantities
54 cameras were sold at some profit and the remaining 6 were returned, incurring a loss => the quantity ratio is 54 : 6 = 9 : 1
The 54 cameras were sold at 20% profit and the 6 were effectively sold at 50% loss

Thus, the net percent profit or loss can be simply obtained by a weighted average (note that all cameras have the same CP):

\(\frac{[9 * 20 + 1 * (-50)]}{(9 + 1)}\) = 13%

Answer B
GMAT Club Legend
GMAT Club Legend
Joined: 03 Jun 2019
Posts: 5343
Own Kudos [?]: 3964 [1]
Given Kudos: 160
Location: India
GMAT 1: 690 Q50 V34
WE:Engineering (Transportation)
Send PM
Re: A photography dealer ordered 60 Model X cameras to be sold [#permalink]
1
Kudos
Given: A photography dealer ordered 60 Model X cameras to be sold for $250 each, which represents a 20 percent markup over the dealer’s initial cost for each camera. Of the cameras ordered, 6 were never sold and were returned to the manufacturer for a refund of 50 percent of the dealer's initial cost.

Asked: What was the dealer's approximate profit or loss as a percent of the dealer’s initial cost for the 60 cameras?

Dealer's initial cost of each camera = $250/1.2

Dealer's initial cost of 60 cameras = $250/1.2 * 60 = $12500

Total sales of 60 cameras = (60-6)*$250 + 6*50%*$250/1.2 = $14125

Percentage profit on 60 cameras = ($14125 - $12500) / $12500 * 100% = 13%

IMO D
avatar
Intern
Intern
Joined: 18 Apr 2015
Posts: 1
Own Kudos [?]: [0]
Given Kudos: 0
Send PM
Re: A photography dealer ordered 60 Model X cameras to be sold [#permalink]
Suppose: what if I want to calculate the number of the total profit first. How can I get it? I tried to get it by "54^250/1.2. Is this is the correct way? Pls enlighten. Thanks in advance.
avatar
Intern
Intern
Joined: 02 Feb 2016
Posts: 1
Own Kudos [?]: [0]
Given Kudos: 4
Send PM
Re: A photography dealer ordered 60 Model X cameras to be sold [#permalink]
Does this question bother anyone else in formatting terms? Perhaps I'm misinterpreting, but let me explain:

"What was the dealer's approximate profit or loss as a percent of the dealer's initial cost for the 60 cameras?"

Profit = Revenue - Costs

Thus, the question is looking for (Revenue - Cost) / Cost

In number terms this would look like, [54*(250) + 6*(250)*(4/5)*(1/2) - 600*(250)*(4/5)]/[(60)*(250)*(4/5)]x100 ~~ 10.7%

I'd appreciate any help in clearing up this misunderstanding!
avatar
Manager
Manager
Joined: 18 Feb 2015
Posts: 71
Own Kudos [?]: 567 [0]
Given Kudos: 15
Send PM
Re: A photography dealer ordered 60 Model X cameras to be sold [#permalink]
MacFauz wrote:
carcass wrote:
A photography dealer ordered 60 Model X cameras to be sold for$250 each, whichrepresents a 20 percent markup over the dealer's initialcost for each camera.
Of the cameras ordered, 6 were never sold and were returned to the manufacturer for a refund of 50 percent of the dealer's initial cost. What was the dealer's approximate profit or loss as a percent of the dealer's initial cost for the 60 cameras?

(A) 7% loss

(B) 13% loss

(C) 7% profit

(D) 13% profit

(E) 15% profit

have fun. is not difficult to conceptualize but is difficult because if you are not organized and clear you are get in trouble.

I would not show the OA but the rules are stringent. either way try on your own :)


Cost to dealer for 60 cameras = \(\frac{250}{1.2}*60\) = $12,500

Revenue for 54 cameras = 54*250 = $13,500

Revenue from 6 cameras = \(\frac{250}{2.4}*6\) = $625

Total Revenue = $14,125

Profit percent = \(\frac{14125-12500}{12500}\) = 13%

Answer is D.

Otherwise, If one were short on time and I had to guess, One can eliminate A,B & C by looking at the problem itself and improve my chances of guessing the right answer to 50% between D & E.

Kudos Please... If my post helped.


Hey all,
Why do we divide 250/2.4? Where does 2.4 come from?
Thanks!
Tutor
Joined: 16 Oct 2010
Posts: 14817
Own Kudos [?]: 64899 [0]
Given Kudos: 426
Location: Pune, India
Send PM
Re: A photography dealer ordered 60 Model X cameras to be sold [#permalink]
Expert Reply
HarveyKlaus wrote:
MacFauz wrote:
carcass wrote:
A photography dealer ordered 60 Model X cameras to be sold for$250 each, whichrepresents a 20 percent markup over the dealer's initialcost for each camera.
Of the cameras ordered, 6 were never sold and were returned to the manufacturer for a refund of 50 percent of the dealer's initial cost. What was the dealer's approximate profit or loss as a percent of the dealer's initial cost for the 60 cameras?

(A) 7% loss

(B) 13% loss

(C) 7% profit

(D) 13% profit

(E) 15% profit

have fun. is not difficult to conceptualize but is difficult because if you are not organized and clear you are get in trouble.

I would not show the OA but the rules are stringent. either way try on your own :)


Cost to dealer for 60 cameras = \(\frac{250}{1.2}*60\) = $12,500

Revenue for 54 cameras = 54*250 = $13,500

Revenue from 6 cameras = \(\frac{250}{2.4}*6\) = $625

Total Revenue = $14,125

Profit percent = \(\frac{14125-12500}{12500}\) = 13%

Answer is D.

Otherwise, If one were short on time and I had to guess, One can eliminate A,B & C by looking at the problem itself and improve my chances of guessing the right answer to 50% between D & E.

Kudos Please... If my post helped.


Hey all,
Why do we divide 250/2.4? Where does 2.4 come from?
Thanks!


Cost to dealer = 250/1.2 (because 250 is the selling price which 20% markup)

Revenue from the 6 cameras is half the cost price. This is = (250/1.2)*(1/2) = 250/2.4

P.S. - Saw your post later Engr2012
Manager
Manager
Joined: 22 Apr 2016
Status:On a 600-long battle
Posts: 123
Own Kudos [?]: 102 [0]
Given Kudos: 392
Location: Hungary
Concentration: Accounting, Leadership
Schools: Erasmus '19
GMAT 1: 410 Q18 V27
GMAT 2: 490 Q35 V23
Send PM
Re: A photography dealer ordered 60 Model X cameras to be sold [#permalink]
VeritasPrepKarishma wrote:
pgmat wrote:
A photography dealer ordered 60 Model X cameras to be sold for $250 each, which represents a 20 percent markup over the dealer’s initial cost for each camera. Of the cameras ordered, 6 were never sold and were returned to the manufacturer for a refund of 50 percent of the dealer's initial cost. What was the dealer's approximate profit or loss as a percent of the dealer’s initial cost for the 60 cameras?

A. 7% loss
B. 13% loss
C. 7% profit
D. 13% profit
E. 15% profit


Use weighted avgs for a quick solution:

On 54 cameras, the dealer made a 20% profit and on 6 cameras, he made a 50% loss. Cost Price of 54 cameras:cost price of 6 cameras = 54:6 = 9:1

Avg Profit/Loss = (.2*9 + (-.5)*1)/10 = 1.3/10 = .13 = 13% profit

For more on weighted avgs, check: https://www.gmatclub.com/forum/veritas-prep-resource-links-no-longer-available-399979.html#/2011/03 ... -averages/


VeritasPrepKarishma, I think you method is pretty sweet - I just used another weighted average looking formula you shared in your blog:

\(\cfrac { w1 }{ w2 } :=\cfrac { \left( A2-Aavg \right) }{ \left( Aavg-A1 \right) } =\\ =\cfrac { 1 }{ 9 } =\cfrac { \left( 1.2-Aavg \right) }{ \left( Aavg-0.5 \right) } \\ =Aavg-0.5=9\left( 1.2-Aavg \right) \\ =Aavg-0.5=10.8-9Avg\\ =Aavg+9Aavg=10.8+0.5\\ =10Aavg=11.3\\ =Aavg=\cfrac { 11.3 }{ 10 } \\ Aavg=1.13\)

13% profit

I'm a big fan of the scaled method but I can't figure out how to set it up as to find the 'combined average'.
GMAT Club Bot
Re: A photography dealer ordered 60 Model X cameras to be sold [#permalink]
   1   2   3   
Moderators:
Math Expert
92902 posts
Senior Moderator - Masters Forum
3137 posts

Powered by phpBB © phpBB Group | Emoji artwork provided by EmojiOne